Helicidades en la aniquilación electrón-positrón

Considere el límite sin masa de un proceso en el que un par electrón-positrón se aniquila en un fotón virtual; el estado final no importa. Si el electrón no tiene masa (o si la energía es lo suficientemente alta), la helicidad y la quiralidad se vuelven iguales y se conservan. Mi problema es que obtengo resultados contradictorios: las matemáticas dicen que la amplitud es distinta de cero solo cuando el electrón y el positrón tienen la misma helicidad, mientras que todos los libros sobre el tema (y el sentido común físico) afirman lo contrario.

La amplitud es proporcional a v ¯ γ m tu , dónde tu es el espinor del electrón y v el de positrones. Vayamos al marco del centro de masa, y tomemos el momento del electrón como pag m = ( pag , 0 , 0 , pag ) y el positrón va a ser pag m = ( pag , 0 , 0 , pag ) . Usando la base de Dirac, tengo los siguientes espinores de helicidad definidos (siguiendo el artículo de Wikipedia sobre espinores ):

tu R = ( 1 0 1 0 )       tu L = ( 0 1 0 1 )

v R = ( 0 1 0 1 )       v L = ( 1 0 1 0 )

Supongamos que el electrón tiene helicidad positiva y el positrón tiene helicidad negativa; en otras palabras, ambos tienen giro hacia arriba a lo largo del eje z. Libros como Modern Particle Physics de Thomson o Quarks and Leptons de Halzen y Martin dicen que la aniquilación debería tener lugar en este caso, y tiene sentido: el estado inicial tiene giro total 1, justo lo que necesitas para crear el fotón virtual.

El problema es que puedo calcular v ¯ L γ m tu R explícitamente, y obtengo cero. Incluso puedo mostrarlo de forma abstracta: Definiendo PAG R = 1 2 ( 1 + γ 5 ) y PAG L = 1 2 ( 1 γ 5 ) y notando que tu R = PAG R tu R y así sucesivamente, se puede demostrar de manera bastante general que v ¯ γ m tu desaparece a menos que ambos espinores tengan la misma helicidad.

¿Que esta pasando aqui? Mi mejor suposición es que, de alguna manera, las asignaciones de helicidad para las antipartículas se invierten, pero no veo cómo puede ser eso: solo seguí el artículo de Wikipedia y todos los libros que pude encontrar, sin mencionar que verifiqué que mis espinores satisfacen la ecuación de Dirac con el momento adecuado, y que los espines son correctos y que PAG L v R = 0 y PAG R v L = 0 .

Solo comprobando porque soy demasiado perezoso para comprobar los cálculos yo mismo en este momento: Te acordaste de que hay un γ 0 en la definición del conjugado v ¯ , ¿bien?
@ACuriousMind: Sí, lo hice. Como dije, hay una prueba que no se basa en cálculos concretos.

Respuestas (2)

No es cierto que " v ¯ L γ m tu R es cero". Mientras que tu R PAG R tu , si revisas cuidadosamente encontrarás que v ¯ L v ¯ PAG L . Y si "tomas PAG L al otro lado de la γ m " utilizando las relaciones anticonmutación usuales, se obtiene PAG R . Y por supuesto, PAG R 2 = PAG R .

El problema puede haber sido originalmente causado por olvidar que v ¯ L tiene el signo opuesto para γ 5 en los operadores de proyección que tu ¯ L .

Lo dices en serio PAG L y PAG R se invierten cuando se actúa sobre v ? Eso no me sorprendería, excepto que verifiqué con la ecuación de Dirac para asegurarme de que mis positrones tienen un momento z negativo, y verifiqué con la matriz de espín. Σ = diagnóstico ( σ , σ ) asegurarse v L ha girado y v R ha girado hacia abajo. Y encaja con el hecho de que PAG R v L = 0 . Entonces, ¿cómo se pueden invertir los proyectores de helicidad?
>Quieres decir eso PAG L y PAG R se invierten cuando se actúa sobre v ? Si, absolutamente. Véase, por ejemplo, la Tabla 9.2 de la "Introducción a las partículas elementales" de Griffiths, 2ª edición.
Tienes toda la razón, pero ¿cómo reconcilio esto con mi cálculo del impulso de giro? Estás diciendo que lo que he llamado v L de hecho tiene helicidad hacia la derecha, pero puedo comprobar que Σ 3 v L = v L , lo que significa que ha girado y que pag v L = 0 , que según mi definición de pag significa que su momento z es negativo. Entonces, ¿por qué tiene helicidad hacia la derecha? ¿Tiene que ver con la inversión del giro y/o del impulso? (Ahora entiendo por qué la gente abandonó todo el asunto de "las antipartículas son partículas que retroceden en el tiempo")
Sí, la helicidad invertida tiene que ver con el momento "invertido de lo que esperas" para las antipartículas. Otra forma de convencerse a sí mismo podría ser usar el operador de conjugación de carga en los espinores de partículas para obtener los espinores de antipartículas, cuestión que se explica en el texto de Halzen y Martin.

Tu problema es cuando consideras

v = ( 0 1 0 1 )
como diestro (quiralidad o helicidad). En realidad, está asociado a un componente zurdo. Considere el operador de helicidad:
h ^ = 1 | pag | Σ . pag ^
con
Σ = ( σ 0 0 σ )
( σ siendo las matrices de Pauli) y:
pag ^ = i
El v -los espinores están asociados a un término de propagación mi i ( mi t pag . X ) (el signo en la exponencial es opuesto al caso de tu -espinor). Por lo tanto, aplicando h ^ a todo el término ψ = v mi i ( mi t pag . X ) , da:
h ^ ψ = 1 | pag | Σ v . ( pag ) mi i ( mi t pag . X )
Como elegiste un impulso a lo largo del eje z: pag = | pag | tu z , tenemos:
h ^ ψ = 1 | pag | Σ z v | pag | mi i ( mi t pag . X ) = ( 1 0 0 0 0 1 0 0 0 0 1 0 0 0 0 1 ) ( 0 1 0 1 ) mi i ( mi t pag . X ) = ψ
De este modo ψ es zurdo! tu espinor v está asociado a una antipartícula levógira.